3
$\begingroup$

First I tried to use integration: $$y=\lim_{n\to\infty}\frac{a^n}{n!}=\lim_{n\to\infty}\frac{a}{1}\cdot\frac{a}{2}\cdot\frac{a}{3}\cdots\frac{a}{n}$$ $$\log y=\lim_{n\to\infty}\sum_{r=1}^n\log\frac{a}{r}$$ But I could not express it as a riemann integral. Now I am thinking about sandwich theorem.

$$\frac{a}{n!}=\frac{a}{1}\cdot\frac{a}{2}\cdot\frac{a}{3}\cdots\frac{a}{t} \cdot\frac{a}{t+1}\cdot\frac{a}{t+2}\cdots\frac{a}{n}=\frac{a}{t!}\cdot\frac{a}{t+1}\cdot\frac{a}{t+2}\cdots\frac{a}{n}$$ Since $\frac{a}{t+1}>\frac{a}{t+2}>\frac{a}{t+1}>\cdots>\frac{a}{n}$ $$\frac{a^n}{n!}<\frac{a^t}{t!}\cdot\big(\frac{a}{t+1}\big)^{n-t}$$ since $\frac{a}{t+1}<1$, $$\lim_{n\to\infty}\big(\frac{a}{t+1}\big)^{n-t}=0$$ Hence, $$\lim_{n\to\infty}\frac{a^t}{t!}\big(\frac{a}{t+1}\big)^{n-t}=0$$ And by using sandwich theorem, $y=0$. Is this correct?

$\endgroup$
8
  • 1
    $\begingroup$ It looks like you're missing some exponents on your $a$s? $\endgroup$ Aug 18, 2015 at 17:39
  • 1
    $\begingroup$ Your reasoning is correct but you put the $<$ sign the wrong way and left out some exponents. $\endgroup$ Aug 18, 2015 at 17:44
  • $\begingroup$ You seem to be assuming that $a > 0$. Is it true? $\endgroup$
    – user169852
    Aug 18, 2015 at 17:44
  • $\begingroup$ @Bungo, good observation. I think the reasoning will still work though by sandwiching between a positive and a negative exponential. $\endgroup$ Aug 18, 2015 at 17:45
  • $\begingroup$ Thanks for suggesting the edits. $\endgroup$
    – Aditya Dev
    Aug 18, 2015 at 17:46

6 Answers 6

10
$\begingroup$

You missed the lower slice of bread in your sandiwch :)

Assuming that $a>0$, you should make clear that all terms are positive, no matter how obvious seems to be.

You should also write explicitly that there is some $t\in\Bbb N$ such that $t>a$. This is called "Archimedean property" of real numbers.

This would be my proof (I insist, assuming that $a>0$):

There exists some natural $t$ such that $t>a$. Then, for $n> t$ $$0<\frac{a^n}{n!}=\frac{a^t}{t!}\frac{a^{n-t}}{(t+1)\cdots n}<\frac{a^t}{t!}\left(\frac at\right)^{n-t}$$

Since $a/t<1$, the rightmost expression tends to $0$, and hence, by the sandwich theorem $$\frac{a^n}{n!}\to 0$$

Remark: If $a\le 0$, the limit is still $0$, but in this case you should use this fact:

If $a_n$ is a sequence of real numbers such that $\lim |a_n|=0$ then $\lim a_n=0$.

$\endgroup$
8
  • $\begingroup$ Is this a common method for solving such type of limits? $\endgroup$
    – Aditya Dev
    Aug 18, 2015 at 17:58
  • 1
    $\begingroup$ I don't know what I should answer. Small changes in an expression can make limits become much harder or much easier to calculate. For me, there are only three secrets: intuit the result (use calculators if needed), read good math books and practice. $\endgroup$
    – ajotatxe
    Aug 18, 2015 at 18:04
  • $\begingroup$ Why is $t! (t+a) ... n = n!$? $\endgroup$ Aug 18, 2015 at 18:05
  • $\begingroup$ @Mathguy007 , Do you know that $n!=n\cdot(n-1)!$ $\endgroup$
    – Aditya Dev
    Aug 18, 2015 at 18:09
  • 1
    $\begingroup$ @AdityaDev: I guess one way to tackle such things is to learn to compare different rates of growth. Often the geometric sequence or series is very useful in obtaining basic bounds on other sequences or series, which is indeed the key to the method used here. $\endgroup$
    – user21820
    Aug 19, 2015 at 3:17
7
$\begingroup$

This is the shortest proof

$\displaystyle \sum _{n=1} ^{\infty} \frac{a^{n}}{n!}$ converges by ratio test.

Let $x_{n}=\frac{a^{n}}{n!} $

Then the convergence of $\displaystyle \sum _{n=1} ^{\infty }x_{n} $ implies $\lbrace x_{n} \rbrace $ converges to zero

$\endgroup$
1
  • 2
    $\begingroup$ In fact, the ratio test is even easier here than the root test. $\endgroup$ Aug 18, 2015 at 18:05
6
$\begingroup$

What is,

$$\lim_{n \to {\infty}} {{a^n} \over {n!}}$$

Well, for large values of $n$, $n!$ can be evaluated with Stirling's Approximation.

$$\lim_{n \to {\infty}} {{a^n} \over {n!}}=\lim_{n \to {\infty}} {{a^n} \over {\sqrt{2 \pi n} \cdot (n/e)^n}}$$

$${{a^n} \over {\sqrt{2 \pi n} \cdot (n/e)^n}} = {1 \over {\sqrt{2 \pi n}}} \cdot a^n \cdot (e/n)^n = {1 \over {\sqrt{2 \pi n}}} \cdot \left({{a \cdot e } \over {n}} \right)^n$$

The value of the limit is clearly $0$ for any finite $a$, $$\lim_{n \to {\infty}} {{a^n} \over {\sqrt{2 \pi n} \cdot (n/e)^n}} = 0$$ This is because the square root term goes to zero, and because the term inside the parentheses must be less than 1 since $a$ is finite.

Thus,

$$\lim_{n \to {\infty}} {{a^n} \over {n!}}=0$$

$\endgroup$
14
  • $\begingroup$ Thanks for mentioning "Stirling's Approximation". But I dont understand why you neglected $\sqrt{2\pi n}$. $\endgroup$
    – Aditya Dev
    Aug 18, 2015 at 17:53
  • 1
    $\begingroup$ @AdityaDev I already added it back in, it was definitely needed for completeness :) $\endgroup$
    – Zach466920
    Aug 18, 2015 at 17:53
  • $\begingroup$ for a finite $a$, $\big(\frac{a\cdot e}{n}\big)^n$ is like $0^{\infty}$ which is an indeterminate form. $\endgroup$
    – Aditya Dev
    Aug 18, 2015 at 17:54
  • $\begingroup$ @AdityaDev The limit is not indeterminate. $\lim_{n \to {\infty}} 0^n=0$ See wolfram $\endgroup$
    – Zach466920
    Aug 18, 2015 at 17:56
  • 2
    $\begingroup$ I learned something new from this answer :) $\endgroup$
    – Aditya Dev
    Aug 18, 2015 at 18:00
3
$\begingroup$

Let $x_n=\frac{a^n}{n!}$.

$$\left|\frac{x_{n+1}}{x_n}\right|=\frac{\frac{a^{n+1}}{(n+1)!}}{\frac{a^n}{n!}}=\frac{a^{n+1}n!}{a^n (n+1)!}= \frac{a}{n+1}\underset{n\to \infty }{\longrightarrow }0$$

and thus, by $x_n\to 0$ by Ratio test.

$\endgroup$
3
$\begingroup$

You can prove it as follows:

for every $\varepsilon >0$ and $m+1>\left| a \right| $ and if $n$ is big enough then $$0<\left| \frac {a^n}{ n! } \right| =\frac { \left| a \right| }{ 1 } \cdot \frac { \left| a \right| }{ 2 } \cdots \frac { \left| a \right| }{ m } \cdot \frac { \left| a \right| }{ m+1 } \cdots \frac { \left| a \right| }{ n } <\frac { { \left| a \right| }^m }{ m! } { \left( \frac { \left| a \right| }{ m+1 } \right) }^{ n-m }<\varepsilon $$

$\endgroup$
3
  • $\begingroup$ dear downvoter can you explain,where is mistake? $\endgroup$
    – haqnatural
    Aug 18, 2015 at 17:54
  • $\begingroup$ There must be some constraint on $n$ (in terms of $m$, $a$, and $\epsilon$) in order to make this inequality true. $\endgroup$
    – user169852
    Aug 18, 2015 at 18:18
  • $\begingroup$ you are right @Bungo,I forgot about it,and have edited already $\endgroup$
    – haqnatural
    Aug 18, 2015 at 18:28
0
$\begingroup$

Here's an easy way that I'm surprised is not yet here:

$$ \frac{a^n}{n!} = \frac{\overbrace{a\cdots\cdots\cdots\cdots a}}{\underbrace{1\cdot2\cdot3\cdots\,\cdots n}} $$ When $n$ reaches the point of being twice as big as $a,$ then every time you increment $n$ by $1$ after that, you multiply the numerator by $a$ and the denominator by more than $2a,$ so the fraction gets multiplied by something whose absolute value is less than $1/2.$ Multiplying by something less than $1/2$ over and over again will give you a product approaching $0.$

$\endgroup$

Not the answer you're looking for? Browse other questions tagged .